1answer.
Ask question
Login Signup
Ask question
All categories
  • English
  • Mathematics
  • Social Studies
  • Business
  • History
  • Health
  • Geography
  • Biology
  • Physics
  • Chemistry
  • Computers and Technology
  • Arts
  • World Languages
  • Spanish
  • French
  • German
  • Advanced Placement (AP)
  • SAT
  • Medicine
  • Law
  • Engineering
SIZIF [17.4K]
3 years ago
11

If Inga Ingerton's property, valued at $35,000, is assessed at 40% of its value, and the mill levy is 83, then what is Inga's an

nual tax on this property?
Mathematics
1 answer:
Pavel [41]3 years ago
3 0

Answer:

Annual tax = $ 1162

Step-by-step explanation:

Property value = $35000

To calculate annual tax:

Assessed value = $35000 * 0.40

                           = $ 14000

annual tax on assessed value:

Annual tax = $14000 *0.083

                  = $1162

You might be interested in
Write an equation of the hyperbola given that the center is at (2, -3), the vertices are at (2, 3) and (2, - 9), and the foci ar
zavuch27 [327]
Check the picture below.

so, the hyperbola looks like so, clearly a = 6 from the traverse axis, and the "c" distance from the center to a focus has to be from -3±c, as aforementioned above, the tell-tale is that part, therefore, we can see that c = 2√(10).

because the hyperbola opens vertically, the fraction with the positive sign will be the one with the "y" in it, like you see it in the picture, so without further adieu,

\bf \textit{hyperbolas, vertical traverse axis }
\\\\
\cfrac{(y- k)^2}{ a^2}-\cfrac{(x- h)^2}{ b^2}=1
\qquad 
\begin{cases}
center\ ( h, k)\\
vertices\ ( h,  k\pm a)\\
c=\textit{distance from}\\
\qquad \textit{center to foci}\\
\qquad \sqrt{ a ^2 + b ^2}\\
asymptotes\quad  y= k\pm \cfrac{a}{b}(x- h)
\end{cases}\\\\
-------------------------------

\bf \begin{cases}
h=2\\
k=-3\\
a=6\\
c=2\sqrt{10}
\end{cases}\implies \cfrac{[y- (-3)]^2}{ 6^2}-\cfrac{(x- 2)^2}{ b^2}=1
\\\\\\
\cfrac{(y+3)^2}{ 36}-\cfrac{(x- 2)^2}{ b^2}=1
\\\\\\
c^2=a^2+b^2\implies (2\sqrt{10})^2=6^2+b^2\implies 2^2(\sqrt{10})^2=36+b^2
\\\\\\
4(10)=36+b^2\implies 40=36+b^2\implies 4=b^2
\\\\\\
\sqrt{4}=b\implies 2=b\\\\
-------------------------------\\\\
\cfrac{(y+3)^2}{ 36}-\cfrac{(x- 2)^2}{ 2^2}=1\implies \cfrac{(y+3)^2}{ 36}-\cfrac{(x- 2)^2}{ 4}=1

3 0
3 years ago
Someone please help me!!!!!
Katarina [22]

1. Answer (D). By the law of sines, we have \frac{a}{\sin A}=\frac{b}{\sin B}=\frac{c}{\sin C} in any \triangle ABC.

2. Answer (C). The law of cosines, c^2=a^2+b^2-2ab\cos C,accepts up to three sides and an angle as an input.

3. Answer (D). Although this triangle is right, we are not given enough information to uniquely determine its sides and angles - here, we need either one more side or one more angle.

4. Answer (D). Don't get tripped up by answer choice (C) - this is just a rearrangement of the statement of the law of cosines. In choice (D), the signs of a^2 and 2ab\cos C are reversed.

5. Answer (B). By the law of sines, we have \frac{5}{\sin 40^\circ}=\frac{3}{\sin\theta}. Solving gives \theta\approx \boxed{23^\circ},157^\circ. Note that this is the <em>ambiguous (SSA) case</em> of the law of sines, where the given measures could specify one triangle, two triangles, or none at all!

6. Answer (A). Since we know all three sides and none of the angles, starting with the law of sines will not help, so we begin with the law of cosines to find one angle; from there, we can use the law of sines to find the remaining angles.

6 0
3 years ago
Identify the augmented matrix that represents this liner system <br> 2x-3y=4 <br> 5x+7y=39
Lisa [10]

Answer:

its c edg 2020

Step-by-step explanation:

7 0
3 years ago
One number is 6 more than another. the difference between their squares is 192. what is the numbers?
Masteriza [31]
Let the number be x.
then the number is (x + 6)
the difference between their squares is 192
(x + 6)² - x² = 192

expanding we get;

x² + 12x + 36 - x² = 192
12x + 36 = 192
12x = 192 - 36
12x = 156
x = 13

the numbers are 13 and 19.

hope this helps, God bless!
7 0
3 years ago
Wynn is 24 years old and has decided to reduce the number of cups of coffee he buys by 2 cups per day. One cup of coffee typical
Usimov [2.4K]

The total amount he would have at 69 is $343,347.81.

<h3>What is the total amount saved?</h3>

The formula that can be used to determine the future value of the deferred annuity is:

Future value = annuity factor x monthly deposit

Annuity factor = {[(1+r)^n] - 1} / r

Where:

  • r = interest rate = 5.5 /2 = 2.75%
  • n = number of payments = 2 x (69 - 24) = 90

Amount he would save every 6 months:

  • amount saved per day = $2.50 x 2 = $5
  • Amount saved per month : $5 x 30 = $150
  • Amount saved every 6 months = $150 x 6 = $900

Future value : 900 x {[(1.0275^90) - 1] / 0.275}= $343,347.81

To learn more about annuities, please check: brainly.com/question/24108530

#SPJ1

5 0
1 year ago
Other questions:
  • Hey guys! How do you work number 44?! I really need some help, and I would really appreciate it! Thank y’all so much! God Bless!
    12·2 answers
  • All my information is "solve"...no other instructions were given...
    6·2 answers
  • A fair coin is tossed 100 times. What is the probability that more than 55 heads are observed?
    5·2 answers
  • How much larger is the 9 in 92,026 than 954?
    7·1 answer
  • 9. Given the point (6,-8) values of the six trig function.
    5·1 answer
  • Find the product of 3 2/3 as a mixed number
    7·1 answer
  • "which of the numbers below can be classified as an integer" and the options are . 10÷2 , -0.3 ,√20 ,. 0.4​
    8·1 answer
  • Hey lads could you please check out my channel on you.tube please leave a like and subscribe it would help a lot much love stay
    12·1 answer
  • He distance between Miami and Naples is 107 miles. The distance between Miami and Jacksonville is about three times this distanc
    8·1 answer
  • Jeremy wants to borrow $100 from his brother. His brother agreed but
    14·2 answers
Add answer
Login
Not registered? Fast signup
Signup
Login Signup
Ask question!